Problemi straniei

Vuoi proporre i tuoi esercizi? Qui puoi farlo!!

Moderatore: tutor

Biagio
Messaggi: 535
Iscritto il: 01 gen 1970, 01:00
Località: Piacenza

Messaggio da Biagio »

eccovi alcuni problemi che provengono da gare nazionali(recenti) di paesi olimpiicamente ben più forti dell\'italia:
<BR>
<BR>1. Find the integer solutions to a + b = 1 - c, a<sup>3</sup> + b<sup>3</sup> = 1 - c<sup>2</sup>
<BR>
<BR>2. x, y are positive reals such that x + y = 2. Show that x<sup>3</sup>y<sup>3</sup>(x<sup>3</sup> + y<sup>3</sup>) ≤ 2
<BR>
<BR>3. Three fixed circles pass through the points A and B. X is a variable point on the first circle different from A and B. The line AX meets the other two circles at Y and Z (with Y between X and Z). Show that XY/YZ is independent of the position of X.
<BR>
<BR>4. What is the largest possible number of elements in a subset of {1, 2, 3, ... , 9} such that the sum of every pair (of distinct elements) in the subset is different?
<BR>
<BR>come li avete trovati?
andrea84
Messaggi: 203
Iscritto il: 01 gen 1970, 01:00
Località: Trento

Messaggio da andrea84 »

Forse ora ci sono
<BR>
<BR>1) con un pò di smanettamenti(neanche troppi) arriviamo a :
<BR>
<BR>(a+b)(a^2-ab+b^2+a+b-2)=0
<BR>
<BR>da cui (a,-a,1) soluzione
<BR>Ora abbiamo da risolvere:
<BR>
<BR>a^2-ab+b^2+a+b-2=0 ora usando un metodo che il caro ma_go definisce brutale ed inelegante imponiamo che il discriminante di quell\'equazione di secondo grado in a sia un quadrato perfetto:
<BR>9-3b^2-6b=k^2 (1)
<BR>Ripetiamo il ragionamento per la (1) considerandola in b e otteniamo:
<BR>
<BR>3k^2+t^2=36 (2)
<BR>
<BR>Ora la (2) ri riduce a 3n^2+p^2=4 ( t=3p, k=3n).
<BR>Quindi abbiamo n=0 p=2 n=0 p=-2 n=-1 p=-1
<BR> n=1 p=1 n=1 p=-1 n=-1 p=1
<BR>
<BR>E da qui si trovano le altre soluzioni dell\'equazione (non ho voglia di fare i conti).
<BR>
<BR>So di essermi spiegato malissimo ma vado di fretta
<BR>
<BR>Ciao ciao
<BR>
<BR>[ Questo Messaggio è stato Modificato da: andrea84 il 05-04-2004 15:21 ]<BR><BR>[ Questo Messaggio è stato Modificato da: andrea84 il 05-04-2004 16:04 ]
Andrea 84 alias Brend
Avatar utente
karl
Messaggi: 926
Iscritto il: 01 gen 1970, 01:00

Messaggio da karl »

Ecco la mia soluzione del 2° es.( non so quanto elegante
<BR>visto che fa uso di un po\' di analisi).
<BR>Osserviamo che :
<BR>sqrt(xy)<=(x+y)/2 od anche:xy<=1 e xy positivo
<BR>Poniamo ora xy=t e
<BR>f(t)=(xy)^3*(x^3+y^3)=(t^3)*(8-6t) con t<=1 e t positivo
<BR>Derivando rispetto a t:
<BR>f\'(t)=24t^2*(1-t).
<BR>Come si vede per t<=1 f(t) e\' sempre crescente
<BR>e presenta un massimo (assoluto)= 2 per t=1.
<BR>Ne segue :f(t)<=2 ovvero
<BR>x^3*y^3(x^3+y^3)<=2.
<BR><BR><BR>[ Questo Messaggio è stato Modificato da: karl il 05-04-2004 17:33 ]
Avatar utente
XT
Messaggi: 695
Iscritto il: 01 gen 1970, 01:00
Località: Piacenza

Messaggio da XT »

Come non detto. Ho trovato un erroraccio.<BR><BR>[ Questo Messaggio è stato Modificato da: XT il 05-04-2004 19:11 ]
"Signore, (a+b^n)/n=x, dunque Dio esiste!" (L.Euler)
sprmnt21
Messaggi: 559
Iscritto il: 01 gen 1970, 01:00

Messaggio da sprmnt21 »

3. il rapporto XY/YZ e\' uguale al rapporto tra la distanza dei centri dei rispettivi cerchi. Per la prova basta considerare la proiezione dei centri sulla retta XA.
<BR>
<BR>
<BR>
<BR>
Biagio
Messaggi: 535
Iscritto il: 01 gen 1970, 01:00
Località: Piacenza

Messaggio da Biagio »

bene bene....
<BR>x karl: la tua soluzione funziona, ma è quella che già conoscevo...mi farebbe piacere averne una che usi metodi puramente...elementari <IMG SRC="images/forum/icons/icon_smile.gif">
<BR>
<BR>x andrea: non è che ti va di concluderla? <IMG SRC="images/forum/icons/icon_wink.gif">
<BR>x sprmnt21: idem
<BR>
<BR>
Avatar utente
Boll
Messaggi: 1076
Iscritto il: 01 gen 1970, 01:00
Località: Piacenza

Messaggio da Boll »

forse sto dicendo una cazzata, ne sono quasi sicuro, ma non fustigatemi,nel primo per \"smanettare\", come dice Andrea bisogna porre c diverso da 1, che una soluzione accettabile presi qualsiasi a e b.
<BR>Anche a me era venuta l\'equazione di Andrea, poi mi sono incartato considerando che i numeri dovevano essere dispari!!!
<BR>
"Ma devo prendere una n-upla qualsiasi o una n-upla arbitraria?" (Lui)
sprmnt21
Messaggi: 559
Iscritto il: 01 gen 1970, 01:00

Messaggio da sprmnt21 »

1. le terne (a,-a,1) con a intero sono soluzioni del sistema dato.
<BR>
<BR>Supponendo che c =/= 1 ( a+b =/= 0) si ha che
<BR>
<BR>
<BR>(a+b)^3-3ab(a+b)=1-c^2 cioe\' (1-c)^2-3ab = 1+c <==> c^2-3c=3ab (1).
<BR>
<BR>Un\'altra relazione che deriva dalle equazioni date (dividendo la prima per la seconda membro a membro) e\' la seguente a^2-ab+b^2=1+c da cui segue che 1+c > ab (2). Combinando la (1) e la (2) si ha che 3(1+c)>c^2-3c da cui deriva che c < 7. Dalla (1) deriva pure che c e\' multiplo di 3 quindi i valori per c possono essere 3 e 6.
<BR>Per c=3 si ha che ab=0 e a+b=-2 cioe\' (0,-2,3) e (-2,0,3).
<BR>Per c=6 si ha che ab=6 e a+b=-5 cioe\' (-2,-3,3) e (-3,-2,3).
<BR>
<BR>
<BR>Valori negativi di c non vanno bene perche\' a+b e a^3+b^3 devono avere lo stesso segno.
<BR>
<BR>
<BR>[ Questo Messaggio è stato Modificato da: sprmnt21 il 06-04-2004 11:19 ]<BR><BR>[ Questo Messaggio è stato Modificato da: sprmnt21 il 06-04-2004 11:21 ]
sprmnt21
Messaggi: 559
Iscritto il: 01 gen 1970, 01:00

Messaggio da sprmnt21 »

Si ha che x^3+y^3 = (x+y)^3-3xy(x+y) = 8 - 6xy.
<BR>
<BR>Quindi la relazione da provare, posto xy=z, diventa: z^3(4-3z)=<1.
<BR>
<BR>Dato che x+y=2 allora z=xy=<1, da cui segue che il fattore 4-3z e\' positivo. Pertanto, per la AM-GM, si ha che 1={[z+z+z+(4-3z)]/4}^4 >= z^3(4-3z).
<BR>
<BR>
<BR>
<BR><BR><BR>[ Questo Messaggio è stato Modificato da: sprmnt21 il 06-04-2004 12:01 ]
sprmnt21
Messaggi: 559
Iscritto il: 01 gen 1970, 01:00

Messaggio da sprmnt21 »

<!-- BBCode Quote Start --><TABLE BORDER=0 ALIGN=CENTER WIDTH=85%><TR><TD><font size=-1>Quote:</font><HR></TD></TR><TR><TD><FONT SIZE=-1><BLOCKQUOTE>
<BR>x andrea: non è che ti va di concluderla? <IMG SRC="images/forum/icons/icon_wink.gif">
<BR>x sprmnt21: idem
<BR></BLOCKQUOTE></FONT></TD></TR><TR><TD><HR></TD></TR></TABLE><!-- BBCode Quote End -->
<BR>
<BR>Detti O1,O2 ed O3 i centri dei cerchi ed M1,M2 ed M3 le proiezioni di questi sulla retta X(AY)Z si ha che M1, M2 ed M3 sono i punti medi di AX, AY ed AZ rispettivamente.
<BR>
<BR>XY = XA+AY = 2M1A+2AM2 = 2M1M2; YZ = YA+AZ = 2M2A+2AM3 = 2M2M3.
<BR>
<BR>Si ha che XY/YZ = 2M1M2/(2M2M3) = O1O2/O2O3.
<BR>
<BR>
Biagio
Messaggi: 535
Iscritto il: 01 gen 1970, 01:00
Località: Piacenza

Messaggio da Biagio »

beh, me li avete bruciati quasi tutti, a parte il quarto che forse è il più facile.. <IMG SRC="images/forum/icons/icon_smile.gif">
<BR>
<BR>ps:grazie a sprmnt21 per la soluzione al 2 assolutamente elementare
Avatar utente
Boll
Messaggi: 1076
Iscritto il: 01 gen 1970, 01:00
Località: Piacenza

Messaggio da Boll »

Forse ho una soluzione ancora più elementare del 2.
<BR>Una volta scomposta il polinomio risulta x^3*y^3(x+y)(x^2-xy+y^2)<=2
<BR>Dividendo per 2 risulta x^3*y^3(x^2-xy+y^2)<=1
<BR>Dimostro per assurdo ponendo il polinomio:
<BR>x^3*(2-x)^3(x+y)(x^2-x(2-x)+(2-x)^2)>1
<BR>1) x^3>1 per x>1
<BR>2) (2-x)^3>1 per x<1
<BR>3) 3x^2-6x+4>1 qualunque x preso - singoletto 1
<BR>fato lo schema segni risulta che il polinomio può assumere solo valori maggiori o uguali a 1 quindi è ASSURDO.
<BR>
<BR>Sicuro ho sbagliato qualcosa, era troppo facile. datemi notizie, vi prego.
<BR>
<BR><BR><BR>[ Questo Messaggio è stato Modificato da: Boll il 06-04-2004 14:26 ]
"Ma devo prendere una n-upla qualsiasi o una n-upla arbitraria?" (Lui)
ma_go
Site Admin
Messaggi: 1906
Iscritto il: 01 gen 1970, 01:00

Messaggio da ma_go »

uhm... in effetti carino, l\'ultimo...
<BR>allora, le possibili somme vanno da 3 (= 2+1) a 17 (= 8+9).
<BR>se però ci fossero conteporaneamente 1,2,8 e 9, si avrebbe 1+9 = 2+8, quindi non si possono ottenere entrambe le somme 3 e 17.
<BR>ci devono essere quindi meno di 15 coppie; cioè, detto il massimo, si deve avere n(n-1)/2 < 15, quindi n<6.
<BR>ora il massimo n potrebbe essere 5, ed infatti {1,2,3,5,8} soddisfa le richieste.
Biagio
Messaggi: 535
Iscritto il: 01 gen 1970, 01:00
Località: Piacenza

Messaggio da Biagio »

ok, ora che li avete risolti tutti, eccovi la ciliegina sulla torta...
<BR>
<BR>5. dimostrare che esistono infiniti n naturali tali che sigma(n)/n > sigma(k)/k per ogni k naturale minore di n, essendo sigma(n) la funzione che associa ad n la somma di tutti i suoi divisori (1 ed n compresi).
Avatar utente
Boll
Messaggi: 1076
Iscritto il: 01 gen 1970, 01:00
Località: Piacenza

Messaggio da Boll »

Posso solo affermare con abbastanza sicurezza che tale propietà è verificata per ogni n tale che n=p-1 dove p è un qualsiasi numero primo, ma sulla dimostrazione non ho neanche una vaga idea.<BR><BR>[ Questo Messaggio è stato Modificato da: Boll il 06-04-2004 22:02 ]
"Ma devo prendere una n-upla qualsiasi o una n-upla arbitraria?" (Lui)
Bloccato